← Back to Topics

muscle weakness

Practice targeted AMC-style multiple-choice questions on muscle weakness.

Related Topics

CT, ultrasonography, SPECT and scintigraphy of intra-thyroid parathyroid adenoma
Image by Mnahi Bin Saeedan, Ibtisam Musallam Aljohani, Ayman Omar Khushaim, Salwa Qasim Bukhari, Salahudin Tayeb Elnaas CC BY 4.0 · Source

A 58-year-old woman presents with fatigue, muscle weakness, and abdominal pain. Labs show serum calcium 3.2 mmol/L and PTH 190 pg/mL. Renal function is normal. Imaging studies are shown. Based on the clinical presentation, laboratory findings, and imaging studies, what is the MOST appropriate next step in managing this patient?

Mark this as a high-quality question
Mark this as a poor-quality or problematic question

A 72-year-old man with a history of hypertension, type 2 diabetes, and chronic kidney disease (CKD) presents to his general practitioner for a routine check-up. His current medications include metformin, insulin glargine, amlodipine, and aspirin. His blood pressure today is 150/90 mmHg. Laboratory results show a serum creatinine of 2.5 mg/dL (baseline 2.0 mg/dL), potassium of 5.4 mEq/L, and a urine albumin-to-creatinine ratio (ACR) of 350 mg/g. The GP decides to add an ACE inhibitor to his medication regimen to help manage his blood pressure and proteinuria. One week later, the patient returns complaining of fatigue and muscle weakness. Repeat laboratory testing reveals a serum creatinine of 3.1 mg/dL and a potassium of 6.2 mEq/L. Which of the following is the most appropriate next step in managing this patient?

Mark this as a high-quality question
Mark this as a poor-quality or problematic question

A 68-year-old woman presents with a 2-week history of progressive weakness in her right arm and leg, associated with a tremor in the right hand. She also reports feeling more tired than usual. Her general practitioner noted an elevated TSH (12 mIU/L, reference range 0.4-4.0 mIU/L) on routine blood tests done a month ago, but free T4 was within the normal range (15 pmol/L, reference range 10-20 pmol/L). On examination, she has mild right-sided hemiparesis (MRC grade 4/5) and a coarse tremor in the right hand, worse with action. Reflexes are brisk on the right. The rest of the neurological exam is unremarkable. She has mild bradycardia (55 bpm) but no other signs of overt hypothyroidism. What is the most appropriate initial investigation?

Mark this as a high-quality question
Mark this as a poor-quality or problematic question

A 68-year-old woman is admitted to the hospital with confusion and muscle weakness. Her medications include hydrochlorothiazide for hypertension. Initial laboratory results show: Sodium 120 mmol/L (Normal: 135-145 mmol/L), Potassium 3.8 mmol/L (Normal: 3.5-5.0 mmol/L), Chloride 90 mmol/L (Normal: 95-105 mmol/L). What is the most likely cause of her hyponatremia?

Mark this as a high-quality question
Mark this as a poor-quality or problematic question
CT, ultrasonography, SPECT and scintigraphy of intra-thyroid parathyroid adenoma
Image by Mnahi Bin Saeedan, Ibtisam Musallam Aljohani, Ayman Omar Khushaim, Salwa Qasim Bukhari, Salahudin Tayeb Elnaas CC BY 4.0 · Source

A 58-year-old woman presents with persistent fatigue, muscle weakness, and vague abdominal pain. Her serum calcium is 3.2 mmol/L (2.1-2.6 mmol/L), and PTH is 190 pg/mL (10-65 pg/mL). She denies any history of fractures or kidney stones. She was referred for imaging, as shown. What is the MOST appropriate next step in managing this patient?

Mark this as a high-quality question
Mark this as a poor-quality or problematic question
CT, ultrasonography, SPECT and scintigraphy of intra-thyroid parathyroid adenoma
Image by Mnahi Bin Saeedan, Ibtisam Musallam Aljohani, Ayman Omar Khushaim, Salwa Qasim Bukhari, Salahudin Tayeb Elnaas CC BY 4.0 · Source

A 58-year-old woman presents with persistent fatigue, muscle weakness, and vague abdominal pain. Her serum calcium is 3.2 mmol/L (2.1-2.6 mmol/L), and PTH is 190 pg/mL (10-65 pg/mL). Renal function is normal. She denies any family history of endocrine disorders. Imaging studies are performed, as shown. What is the MOST appropriate next step in managing this patient?

Mark this as a high-quality question
Mark this as a poor-quality or problematic question
feedback